www.vorhilfe.de
- Förderverein -
Der Förderverein.

Gemeinnütziger Verein zur Finanzierung des Projekts Vorhilfe.de.
Hallo Gast!einloggen | registrieren ]
Startseite · Mitglieder · Impressum
Forenbaum
^ Forenbaum
Status VH e.V.
  Status Vereinsforum

Gezeigt werden alle Foren bis zur Tiefe 2

Navigation
 Startseite...
 Suchen
 Impressum
Das Projekt
Server und Internetanbindung werden durch Spenden finanziert.
Organisiert wird das Projekt von unserem Koordinatorenteam.
Hunderte Mitglieder helfen ehrenamtlich in unseren moderierten Foren.
Anbieter der Seite ist der gemeinnützige Verein "Vorhilfe.de e.V.".
Partnerseiten
Weitere Fächer:

Open Source FunktionenplotterFunkyPlot: Kostenloser und quelloffener Funktionenplotter für Linux und andere Betriebssysteme
Forum "Lineare Algebra - Moduln und Vektorräume" - Moduln isomorph
Moduln isomorph < Moduln/Vektorraum < Lineare Algebra < Hochschule < Mathe < Vorhilfe
Ansicht: [ geschachtelt ] | ^ Forum "Lineare Algebra - Moduln und Vektorräume"  | ^^ Alle Foren  | ^ Forenbaum  | Materialien

Moduln isomorph: Tipp
Status: (Frage) beantwortet Status 
Datum: 11:24 Mi 20.06.2012
Autor: chesn

Aufgabe
Es seien $ R $ ein euklidischer Ring, $ [mm] q\in [/mm] R $ ein irreduzibles Element sowie $ m,n [mm] \in \IN [/mm] $ mit $ m > n $.
Zeigen Sie, dass die beiden R-Moduln $ [mm] q^{n}R/q^{m}R [/mm] $ und $ [mm] R/q^{m-n}R [/mm] $ isomorph sind.

Hallo! Bräuchte etwas Hilfe bei der Aufgabe, da ich mir noch recht unsicher bin... Meine Ideen:

Es gilt $ [mm] |q^{n}R/q^{m}R|=\bruch{q^m}{q^n}=q^{m-n}=|R/q^{m-n}R| [/mm] $ d.h. beide Moduln haben die gleiche Anzahl von Elementen.

Jetzt habe ich versucht einen Homomorphismus zu konstruieren, der den einen auf den anderen Modul abbildet. Hier mein improvisierter Ansatz, bei dem ich Hilfe benötige:

[mm] q^{n}R/q^{m}R=\{q^n, 2q^n, 3q^n, ... ,kq^n\} [/mm] wobei [mm] k*q^n=q^m \gdw k=q^{m-n} [/mm] und $ [mm] q^m\equiv [/mm] 0 $

[mm] R/q^{m-n}R=\{1,2,3,...,k\} [/mm] wobei $ [mm] k=q^{m-n}\equiv [/mm] 0 $

Jetzt definiere ich $ [mm] \pi [/mm] : \ \  [mm] q^{n}R/q^{m}R \to R/q^{m-n}R [/mm] \ \ , \ \ [mm] r*q^n \mapsto [/mm] r$

Dafür könnte ich jetzt die Eigenschaften eines Modulhomomorphismus nachweisen.

Und zur Bijektivität: Da beide Moduln gleichmächtig sind, bleibt nur die Injektivität zu zeigen:

[mm] kern(\phi)=\{ r*q^n \ | \ r=0 \ mod \ q^{m-n}\}=\{k*q^n\} [/mm]

Es ist [mm] k*q^n=0 [/mm] mod [mm] q^m [/mm] und k=0 mod [mm] q^{m-n} [/mm] also ist der Kern trivial und [mm] \pi [/mm] somit injektiv. => [mm] \pi [/mm] bijektiv.

Wäre nett wenn mich jemand korrigieren könnte! :)

Vielen Dank schonmal und lieben Gruß,
chesn

        
Bezug
Moduln isomorph: Mitteilung
Status: (Mitteilung) Reaktion unnötig Status 
Datum: 14:29 Mi 20.06.2012
Autor: chesn

Oder liege ich ganz daneben? Wäre nett wenn noch jemand was dazu sagen könnte!

Vielen Dank!

Gruß,
chesn

Bezug
        
Bezug
Moduln isomorph: Antwort
Status: (Antwort) fertig Status 
Datum: 08:56 Do 21.06.2012
Autor: hippias


> Es seien [mm]R[/mm] ein euklidischer Ring, [mm]q\in R[/mm] ein irreduzibles
> Element sowie [mm]m,n \in \IN[/mm] mit [mm]m > n [/mm].
>  Zeigen Sie, dass die
> beiden R-Moduln [mm]q^{n}R/q^{m}R[/mm] und [mm]R/q^{m-n}R[/mm] isomorph
> sind.
>  Hallo! Bräuchte etwas Hilfe bei der Aufgabe, da ich mir
> noch recht unsicher bin... Meine Ideen:

Das haut alles nicht so richtig hin.

>  
> Es gilt
> [mm]|q^{n}R/q^{m}R|=\bruch{q^m}{q^n}=q^{m-n}=|R/q^{m-n}R|[/mm] d.h.
> beide Moduln haben die gleiche Anzahl von Elementen.

Das kann man so nicht machen; beachte im Uebrigen die Moeglichkeit, dass es sich um unendliche Mengen handelt.

>  
> Jetzt habe ich versucht einen Homomorphismus zu
> konstruieren, der den einen auf den anderen Modul abbildet.
> Hier mein improvisierter Ansatz, bei dem ich Hilfe
> benötige:
>  
> [mm]q^{n}R/q^{m}R=\{q^n, 2q^n, 3q^n, ... ,kq^n\}[/mm] wobei
> [mm]k*q^n=q^m \gdw k=q^{m-n}[/mm] und [mm]q^m\equiv 0[/mm]
>  
> [mm]R/q^{m-n}R=\{1,2,3,...,k\}[/mm] wobei [mm]k=q^{m-n}\equiv 0[/mm]
>  
> Jetzt definiere ich [mm]\pi : \ \ q^{n}R/q^{m}R \to R/q^{m-n}R \ \ , \ \ r*q^n \mapsto r[/mm]
>

Es handelt sich um Restklassen: sieh Dir die Definition eines Faktormoduls nocheinmal an. Die Elemente von [mm] $q^{n}R/q^{m}R$ [/mm] haben die Gestalt [mm] $q^{n}x+ q^{m}R$, $x\in [/mm] R$; dies ist die Restklasse, die [mm] $q^{n}x$ [/mm] enthaelt bezueglich der Aequivalenzrelation [mm] $x\equiv y:\iff x-y\in q^{n}R$. [/mm]

> Dafür könnte ich jetzt die Eigenschaften eines
> Modulhomomorphismus nachweisen.
>
> Und zur Bijektivität: Da beide Moduln gleichmächtig sind,
> bleibt nur die Injektivität zu zeigen:
>  
> [mm]kern(\phi)=\{ r*q^n \ | \ r=0 \ mod \ q^{m-n}\}=\{k*q^n\}[/mm]
>  
> Es ist [mm]k*q^n=0[/mm] mod [mm]q^m[/mm] und k=0 mod [mm]q^{m-n}[/mm] also ist der
> Kern trivial und [mm]\pi[/mm] somit injektiv. => [mm]\pi[/mm] bijektiv.
>  
> Wäre nett wenn mich jemand korrigieren könnte! :)

Nirgendwo sind die speziellen EIgenschaften von $R$ und $q$ eingegangen. $R$ ist ein belibiger euklidischer Ring und nicht notwendig [mm] $=\IZ$. [/mm]

>  
> Vielen Dank schonmal und lieben Gruß,
>  chesn


Bezug
                
Bezug
Moduln isomorph: Frage (beantwortet)
Status: (Frage) beantwortet Status 
Datum: 09:43 Do 21.06.2012
Autor: chesn

Hallo! Vielen Dank erstmal für deine Antwort. Leider komme ich alleine nicht so wirklich weiter.

Läuft es denn überhaupt darauf hinaus, dass ich einen Homomorphismus konstruieren und dessen Bijektivität zeigen muss, oder kann ich auch anders argumentieren?

Habe mir auch Gedanken gemacht, wie ich eingehen lasse, dass R ein euklidischer Ring ist und q irreduzibel, aber auch da komme ich nicht so wirklich drauf.

Hast du evtl. noch einen Ansatz oder Tipp für mich? :)

Vielen Dank und lieben Gruß,
chesn

Bezug
                        
Bezug
Moduln isomorph: Antwort
Status: (Antwort) fertig Status 
Datum: 10:03 Do 21.06.2012
Autor: hippias


> Hallo! Vielen Dank erstmal für deine Antwort. Leider komme
> ich alleine nicht so wirklich weiter.
>  
> Läuft es denn überhaupt darauf hinaus, dass ich einen
> Homomorphismus konstruieren und dessen Bijektivität zeigen
> muss, oder kann ich auch anders argumentieren?
>  
> Habe mir auch Gedanken gemacht, wie ich eingehen lasse,
> dass R ein euklidischer Ring ist und q irreduzibel, aber
> auch da komme ich nicht so wirklich drauf.
>  
> Hast du evtl. noch einen Ansatz oder Tipp für mich? :)

Ja, klar: Ich wuerde eine Abbildung [mm] $\phi: R\to q^{n}R/q^{m}R$ [/mm] durch [mm] $x\mapsto q^{n}x+q^{m} [/mm] R$ definieren und versuchen nachzuweisen, dass es ein $R$-Modulhomomorphismus ist. Ferner, dass er surjektiv und sein Kern $= [mm] q^{m-n}R$ [/mm] ist; mit einem der Homomorphiesaetze folgt dann die Behauptung. Im Uebrigen glaube ich fast, dass es genuegt $q$ als Nichtnullteiler vorauszusetzen.

>  
> Vielen Dank und lieben Gruß,
>  chesn


Bezug
                                
Bezug
Moduln isomorph: Frage (beantwortet)
Status: (Frage) beantwortet Status 
Datum: 11:44 Do 21.06.2012
Autor: chesn

Tausen Dank erstmal! Das hat sehr geholfen.

Also ich muss zeigen dass die Abbildung [mm] \Phi [/mm] ein Homomorphismus ist:

Ich würde sagen die Wohldefiniertheit folgt direkt, da für $ [mm] x_1,x_2\in [/mm] R $ mit [mm] x_1=x_2 [/mm] direkt folgt das [mm] \Phi(x_1)=\Phi(x_2) [/mm] oder gibt es da was zu zeigen?

[mm] \Phi(x_1+x_2)=\Phi(x_1)+\Phi(x_2) [/mm] und [mm] \Phi(r*x)=r*\Phi(x) [/mm] ist klar.

$ [mm] kern(\Phi)=\{x\in R \ | \ q^n*x=q^mR\} [/mm] => q^nx=q^mR [mm] \gdw x=q^{m-n}R [/mm] $ also ist  $ [mm] kern(\Phi)=\{q^{m-n}R\} [/mm] $

Ist denn da wie du sagst wirklich noch die Surjektivität zu zeigen?
Die Isomorphie folgt doch schon durch Anwendung des Homomorphiesatzes (und damit auch die Surjektivität), oder?

Nach Homomorphiesatz: $ [mm] \Phi: R\to [/mm] q^nR/q^mR $ ist Modulhomomorphismus, dann ist [mm] R/kern(\Phi)=R/q^{m-n}R [/mm] isomorph zu [mm] bild(\Phi)=q^nR/q^mR [/mm]

Passt das so?? Und vielen Dank nochmal!! :)

Lieben Gruß,
chesn

Bezug
                                        
Bezug
Moduln isomorph: Antwort
Status: (Antwort) fertig Status 
Datum: 08:54 Fr 22.06.2012
Autor: hippias


> Tausen Dank erstmal! Das hat sehr geholfen.
>  
> Also ich muss zeigen dass die Abbildung [mm]\Phi[/mm] ein
> Homomorphismus ist:
>  
> Ich würde sagen die Wohldefiniertheit folgt direkt, da
> für [mm]x_1,x_2\in R[/mm] mit [mm]x_1=x_2[/mm] direkt folgt das
> [mm]\Phi(x_1)=\Phi(x_2)[/mm] oder gibt es da was zu zeigen?

Nein, da ist nichts weiter zu zeigen.

>  
> [mm]\Phi(x_1+x_2)=\Phi(x_1)+\Phi(x_2)[/mm] und [mm]\Phi(r*x)=r*\Phi(x)[/mm]
> ist klar.
>  
> [mm]kern(\Phi)=\{x\in R \ | \ q^n*x=q^mR\} => q^nx=q^mR \gdw x=q^{m-n}R[/mm]
> also ist  [mm]kern(\Phi)=\{q^{m-n}R\}[/mm]

Zeige beide Inklusionen separat. Denn fuer die eine Inklusion geht eine Eigenschaft ein, die Du vermutlich uebersehen hast.

>  
> Ist denn da wie du sagst wirklich noch die Surjektivität
> zu zeigen?
>  Die Isomorphie folgt doch schon durch Anwendung des
> Homomorphiesatzes (und damit auch die Surjektivität),
> oder?
>  
> Nach Homomorphiesatz: [mm]\Phi: R\to q^nR/q^mR[/mm] ist
> Modulhomomorphismus, dann ist [mm]R/kern(\Phi)=R/q^{m-n}R[/mm]
> isomorph zu [mm]bild(\Phi)=q^nR/q^mR[/mm]

Ja, aber dass [mm] $bild(\Phi)=q^nR/q^mR$ [/mm] gilt, ist doch noch nachzuweisen.

>  
> Passt das so?? Und vielen Dank nochmal!! :)
>  
> Lieben Gruß,
>  chesn


Bezug
                                                
Bezug
Moduln isomorph: Frage (überfällig)
Status: (Frage) überfällig Status 
Datum: 12:27 Sa 23.06.2012
Autor: triad

Hi,

ich bin leider noch nicht so geübt in Faktormoduln, mag deshalb jemand schauen, ob ich das richtig gemacht habe?

Also zunächst die Linearität:

[mm] \phi(x_1+x_2)=q^n(x_1+x_2)+q^mR=(q^nx_1+q^nx_2)+q^mR=q^nx_1+q^mR [/mm] + [mm] q^nx_2+q^mR=\phi(x_1)+\phi(x_2) [/mm]

[mm] \phi(r*x)=q^n(rx)+q^mR=q^nrx+q^mR=r*(q^nx)+q^mR=r*\phi(x) [/mm]


> > [mm]kern(\Phi)=\{x\in R \ | \ q^n*x=q^mR\} => q^nx=q^mR \gdw x=q^{m-n}R[/mm]
> > also ist  [mm]kern(\Phi)=\{q^{m-n}R\}[/mm]
>  Zeige beide Inklusionen separat. Denn fuer die eine
> Inklusion geht eine Eigenschaft ein, die Du vermutlich
> uebersehen hast.

Das habe ich nicht ganz verstanden, was hier gemacht wird. Ok, [mm] Ker(\phi)=\{x\in R \ | \ q^n*x=q^mR\}, [/mm] da in ein Faktormodul abgebildet wird, wo genau alle Vielfachen von [mm] q^{m}R [/mm] Null sind. Dann durch [mm] q^n [/mm] teilen und Potenzgesetz angewendet liefert [mm] x=q^{m-n}R, [/mm] also besteht der Kern aus allen [mm] $x\in [/mm] R$ mit [mm] $x=q^{m-n}R$. [/mm] Die andere Inklusion wäre doch: Sei [mm] x\in\{q^{m-n}R\} [/mm] mit [mm] x=q^{m-n}R [/mm] und dann kann man das einfach wieder umformen zu [mm] q^n*x=q^mR, [/mm] aber das steht ja schon kürzer in obigem Zitat :/

>  
> >  

> > Ist denn da wie du sagst wirklich noch die Surjektivität
> > zu zeigen?
>  >  Die Isomorphie folgt doch schon durch Anwendung des
> > Homomorphiesatzes (und damit auch die Surjektivität),
> > oder?
>  >  
> > Nach Homomorphiesatz: [mm]\Phi: R\to q^nR/q^mR[/mm] ist
> > Modulhomomorphismus, dann ist [mm]R/kern(\Phi)=R/q^{m-n}R[/mm]
> > isomorph zu [mm]bild(\Phi)=q^nR/q^mR[/mm]
>  Ja, aber dass [mm]bild(\Phi)=q^nR/q^mR[/mm] gilt, ist doch noch
> nachzuweisen.

Haben wir [mm] \phi [/mm] nicht gerade so definiert, dass [mm] Im(\phi)=q^nR/q^mR [/mm] ?

>  >  
> > Passt das so?? Und vielen Dank nochmal!! :)
>  >  
> > Lieben Gruß,
>  >  chesn
>  


Bezug
                                                        
Bezug
Moduln isomorph: Mitteilung
Status: (Mitteilung) Reaktion unnötig Status 
Datum: 10:50 So 24.06.2012
Autor: chesn

Die Homomorphismuseigenschaften habe ich auch so, das sollte passen...

beim Rest bin ich mir auch noch nicht ganz schlüssig, vllt. kann ja noch mal jemand was dazu sagen?! :)

Lieben Gruß
chesn

Bezug
                                                        
Bezug
Moduln isomorph: Fälligkeit abgelaufen
Status: (Mitteilung) Reaktion unnötig Status 
Datum: 13:20 Mo 25.06.2012
Autor: matux

$MATUXTEXT(ueberfaellige_frage)
Bezug
Ansicht: [ geschachtelt ] | ^ Forum "Lineare Algebra - Moduln und Vektorräume"  | ^^ Alle Foren  | ^ Forenbaum  | Materialien


^ Seitenanfang ^
ev.vorhilfe.de
[ Startseite | Mitglieder | Impressum ]